- Fri Jan 21, 2011 12:00 am
#23626
Complete Question Explanation
Strengthen—Principle. The correct answer choice is (B)
The argument in the stimulus is formulated as follows:
Premise: Some scientists have expressed reservations about quantum theory because of
its counterintuitive consequences.
Premise: But despite rigorous attempts to show that quantum theory’s predictions were
inaccurate, they were shown to be accurate within the generally accepted
statistical margin of error.
Conclusion: These results, which have not been equaled by quantum theory’s competitors,
warrant acceptance of quantum theory.
The argument uses the “some people say” construction. As some scientists
are said to have reservations about quantum theory, you can predict that the author will conclude that
they should not have these reservations, and indeed this occurs in the last sentence of the stimulus.
The author’s reason for concluding that quantum theory should be accepted is that all attempts to
disprove the theory have failed. From the Flaw in the Reasoning section we know this is an error in
the use of evidence: lack of evidence against a position is mistakenly taken to prove that position
is true. The question stem asks you to identify a principle that could help to justify this form of
erroneous reasoning.
Answer choice (A): The author’s reasoning is not based on “fewer counterintuitive consequences,”
but on the fact that the consequences have not been disproven. Hence, this principle would not help
the reasoning and this answer is incorrect.
Answer choice (B): This is the correct answer choice. If this principle is accepted, then the argument
in the stimulus is strengthened. The principle, when slightly restated, asserts that “if a scientific
theory has been subjected to serious attempts to disprove it and has withstood them, then it should
be accepted.” We know from the stimulus that the theory has been subject to serious attempts to
disprove it and it has withstood them. According to the principle, this justifies the conclusion that the
theory should be accepted.
Answer choice (C): The conclusion of the argument in the stimulus is that the theory should be
accepted. Using the principle in this answer choice would lead one to conclude that the consequences
of the theory should not be considered counterintuitive. Since this result differs from the conclusion,
this answer choice is incorrect.
Answer choice (D): This answer, when rephrased using the Unless Equation, reads “if a theory
should be rejected, then it has been subjected to serious attempts to disprove it.” The scenario in the
stimulus meets the necessary condition, but that has no impact on whether the sufficient occurs or
does not occur.
Answer choice (E): This is the Mistaken Reversal of answer choice (B). Applying this principle
fails to justify the argument since only the necessary condition is met, and that does not prove the
sufficient condition (in this case, that the theory should be accepted) will occur. Hence, this answer
choice is incorrect.
Strengthen—Principle. The correct answer choice is (B)
The argument in the stimulus is formulated as follows:
Premise: Some scientists have expressed reservations about quantum theory because of
its counterintuitive consequences.
Premise: But despite rigorous attempts to show that quantum theory’s predictions were
inaccurate, they were shown to be accurate within the generally accepted
statistical margin of error.
Conclusion: These results, which have not been equaled by quantum theory’s competitors,
warrant acceptance of quantum theory.
The argument uses the “some people say” construction. As some scientists
are said to have reservations about quantum theory, you can predict that the author will conclude that
they should not have these reservations, and indeed this occurs in the last sentence of the stimulus.
The author’s reason for concluding that quantum theory should be accepted is that all attempts to
disprove the theory have failed. From the Flaw in the Reasoning section we know this is an error in
the use of evidence: lack of evidence against a position is mistakenly taken to prove that position
is true. The question stem asks you to identify a principle that could help to justify this form of
erroneous reasoning.
Answer choice (A): The author’s reasoning is not based on “fewer counterintuitive consequences,”
but on the fact that the consequences have not been disproven. Hence, this principle would not help
the reasoning and this answer is incorrect.
Answer choice (B): This is the correct answer choice. If this principle is accepted, then the argument
in the stimulus is strengthened. The principle, when slightly restated, asserts that “if a scientific
theory has been subjected to serious attempts to disprove it and has withstood them, then it should
be accepted.” We know from the stimulus that the theory has been subject to serious attempts to
disprove it and it has withstood them. According to the principle, this justifies the conclusion that the
theory should be accepted.
Answer choice (C): The conclusion of the argument in the stimulus is that the theory should be
accepted. Using the principle in this answer choice would lead one to conclude that the consequences
of the theory should not be considered counterintuitive. Since this result differs from the conclusion,
this answer choice is incorrect.
Answer choice (D): This answer, when rephrased using the Unless Equation, reads “if a theory
should be rejected, then it has been subjected to serious attempts to disprove it.” The scenario in the
stimulus meets the necessary condition, but that has no impact on whether the sufficient occurs or
does not occur.
Answer choice (E): This is the Mistaken Reversal of answer choice (B). Applying this principle
fails to justify the argument since only the necessary condition is met, and that does not prove the
sufficient condition (in this case, that the theory should be accepted) will occur. Hence, this answer
choice is incorrect.